Double Integration - Finding the limits

Click For Summary
The discussion focuses on finding the limits of a double integral over the region bounded by the x-axis and the semi-circle defined by x² + y² = 4 for y ≥ 0. Participants express confusion about how to start the problem and clarify that the task is indeed to determine the limits of integration. It is suggested to visualize the region by sketching the graph, which helps in identifying the smallest and largest x-values for the outer integral. The inner integral's limits are determined by the y-values corresponding to a specific x-value. Overall, understanding the geometric representation of the region is essential for solving the integral limits correctly.
NotStine
Messages
25
Reaction score
0

Homework Statement



Find the limits of the integral of z = f(x,y) over the region bounded by the x-axis, and the semi-circle x2 + y2 = 4, y ≥ 0

Homework Equations





The Attempt at a Solution



Where do I start on this question? I can't understand what it is asking.

IbaIdc

Am I supposed to find a,b,c,d limits of the double integrals? If so, can you please point me in the right direction. I am completely lost.
 
Physics news on Phys.org
NotStine said:

Homework Statement



Find the limits of the integral of z = f(x,y) over the region bounded by the x-axis, and the semi-circle x2 + y2 = 4, y ≥ 0

Homework Equations





The Attempt at a Solution



Where do I start on this question? I can't understand what it is asking.
It seems simple enought to me. x2+ y2= 4 gives a circle with center at (0, 0) and radius 4. You are asked to integrate over the top half of that: y> 0.

IbaIdc

Am I supposed to find a,b,c,d limits of the double integrals? If so, can you please point me in the right direction. I am completely lost.
Since the problem specifically said "find the limitsof integration", yes, that is what you are supposed to do! I recommend that you start by drawing the graph.

Remember that, since the final result is a number, the limits of integration on outer integral must be numbers. It will be simplest to integrate over y first then x so the limits of integration must be values of x. What is the smallest x value in this region? That will be the lower limit. What is the largest x value in this region? That will be the upper limit.

The "inner" integral is with respect to y so the limits may be functions of x. Draw a vertical line on your graph representing some value of x. What is the y value of the lower end of that line? That is the lower limit on the integral. What is the y value on the upper end of that line (a function of x)? That is the upper limit on the integral.
 
Ok that is much more clear now. Thank you.
 
Question: A clock's minute hand has length 4 and its hour hand has length 3. What is the distance between the tips at the moment when it is increasing most rapidly?(Putnam Exam Question) Answer: Making assumption that both the hands moves at constant angular velocities, the answer is ## \sqrt{7} .## But don't you think this assumption is somewhat doubtful and wrong?

Similar threads

Replies
3
Views
2K
  • · Replies 16 ·
Replies
16
Views
2K
Replies
2
Views
2K
  • · Replies 9 ·
Replies
9
Views
2K
  • · Replies 17 ·
Replies
17
Views
4K
  • · Replies 19 ·
Replies
19
Views
2K
Replies
24
Views
3K
Replies
10
Views
2K
  • · Replies 2 ·
Replies
2
Views
2K
  • · Replies 18 ·
Replies
18
Views
2K